[Parcial 2016][Ej 2 b)]

[Parcial 2016][Ej 2 b)]

de Maria Jose Reynaldo Argenzio -
Número de respuestas: 6
Buenas una consulta,


En este ejercicio me queda una gran duda si es verdadero o falso, por un lado no existe ninguna valuación que haga verdadero a phi y al mismo tiempo haga verdadero a botton o como se escriban(jaj) pero por otro lado si usas los teoremas de corrección y completitud se me ocurre poder demostrarlo, 

Ayuda, porfa!


Saludos

En respuesta a Maria Jose Reynaldo Argenzio

Re: [Parcial 2016][Ej 2 b)]

de Sebastian Ignacio Rizzo Martin -
Para mi es falsa por el siguiente contra ejemplo: p0-> p0  esta formula es tautologia y esta en H
En respuesta a Sebastian Ignacio Rizzo Martin

Re: [Parcial 2016][Ej 2 b)]

de Maria Jose Reynaldo Argenzio -
si eso pense yo, pero si aplicas completitud y correctitud podes demostrar que es verdad o no? me  confunde porque en el e utilizas esto tambien
En respuesta a Maria Jose Reynaldo Argenzio

Re: [Parcial 2016][Ej 2 b)]

de Sebastian Ignacio Rizzo Martin -
Para mi es falsa xq dice q ninguna proposición de H es tautologia entonces con encontrar una q sea tautologia ya estas negando la afirmacion y podes decir q es FALSA.
En respuesta a Sebastian Ignacio Rizzo Martin

Re: [Parcial 2016][Ej 2 b)]

de Alejandro Jose Rojas Quartino -

Me parece que Majo se refiere al ejercicio 2017 aún.

La pregunta es si la siguiente afirmación es verdadera o falsa justificando:

 (\exists \varphi \in PROP) (\varphi \neq \perp y \varphi \models \perp)

Tomando cualquier contradicción, en particular \varphi = (\alpha \wedge \neg \alpha) se cumple la afirmación.

Estoy bien o le erré como a las chapas?


Saludos!